How Do You Solve a Complex Divergence Problem Using Spherical Coordinates?

  • Thread starter Thread starter [Quadratic]
  • Start date Start date
  • Tags Tags
    Calc 3 Divergence
Click For Summary
The discussion revolves around solving a complex divergence problem using spherical coordinates, specifically focusing on the parameterization of the vector function F(x,y,z) and the setup of the limit and integral. The user expresses confusion about how to handle the dot product and the surface element dS, questioning whether it can be converted to Cartesian coordinates. There are frustrations regarding the disorganization of the professor, who has made errors in exams and assigned problems that were not adequately covered in lectures. Despite these challenges, the professor claims the problem is solvable, and many classmates are struggling with it as well. The conversation highlights the difficulties students face in understanding the application of spherical coordinates in divergence problems.
[Quadratic]
Messages
59
Reaction score
3

Homework Statement


Apologies for the attachment.

XuIvD8u.png

Homework Equations


Limit definition of the divergence as seen in attachment
Volume of a sphere: \frac{4}{3}\pi r^{3}

The Attempt at a Solution


The first thing I did was parameterize the vector function F(x,y,z) = <xy,x,y+z>
My parameterization is as follows:

<br /> x = a+rcos\vartheta sin\varphi \\<br /> y = b+rsin\vartheta sin\varphi \\<br /> z = c+rcos\varphi \\<br /> dS = S_{\varphi} X S_{\vartheta} d\varphi d\vartheta<br />

Setting up the limit and integral:
<br /> lim_{r\rightarrow0}\frac{1}{\frac{4}{3}\pi r^3} \int^{2\pi}_{\vartheta=0} \int^{\pi}_{\varphi=0} &lt;(a + rcos\vartheta sin\varphi)(b + rsin\vartheta sin\varphi),a + rcos\vartheta sin\varphi,rsin\vartheta sin\varphi + c + rcos\varphi&gt; \bullet S_{\varphi} X S_{\vartheta} d\varphi d\vartheta \\<br />

I apologize for the large attachment and my messy latex. Any suggestions to clean it up are welcome. Am I on the right track so far, before I continue? I tried using the Jacobian thinking it would clean up the integrand but I didn't really get anywhere, and my professor told me we are not doing a change of variables here. Thanks in advance.
 
Physics news on Phys.org
You somehow have to perform the dot product. You have F in Cartesian but not dS. I assume you're allowed to put dS in Cartesian? If not, I don't know how you're expected to continue.
 
haruspex said:
You somehow have to perform the dot product. You have F in Cartesian but not dS. I assume you're allowed to put dS in Cartesian? If not, I don't know how you're expected to continue.

I really don't know. My teacher is rather "disorganized" to put it lightly and frequently makes mistakes on our exams. For example our last exam had 12 problems, and 5 of them had pretty critical errors and were not corrected until around an hour into the exam. He also assigns us problems that require methods he skips over in his lectures. Look at me, now I'm just complaining :)

Almost everyone in the class is having trouble with this, and our professor assured us he worked this one out and it is solvable.
 
[Quadratic];4574654 said:
I really don't know. My teacher is rather "disorganized" to put it lightly and frequently makes mistakes on our exams. For example our last exam had 12 problems, and 5 of them had pretty critical errors and were not corrected until around an hour into the exam. He also assigns us problems that require methods he skips over in his lectures. Look at me, now I'm just complaining :)

Almost everyone in the class is having trouble with this, and our professor assured us he worked this one out and it is solvable.

OK, so assume you're allowed to put dS in Cartesian. Have you tried that?
 
Question: A clock's minute hand has length 4 and its hour hand has length 3. What is the distance between the tips at the moment when it is increasing most rapidly?(Putnam Exam Question) Answer: Making assumption that both the hands moves at constant angular velocities, the answer is ## \sqrt{7} .## But don't you think this assumption is somewhat doubtful and wrong?

Similar threads

  • · Replies 20 ·
Replies
20
Views
2K
  • · Replies 4 ·
Replies
4
Views
2K
  • · Replies 3 ·
Replies
3
Views
9K
  • · Replies 4 ·
Replies
4
Views
2K
  • · Replies 4 ·
Replies
4
Views
3K
  • · Replies 8 ·
Replies
8
Views
1K
  • · Replies 8 ·
Replies
8
Views
2K
  • · Replies 4 ·
Replies
4
Views
2K
Replies
3
Views
2K